Having an efficient, attractive subway system makes good economic sense. So, the city needs to purchase new subway c...

Batman on January 1, 2015

Fundamental question

I got right answer for this question though, I can't help but wonder about the right way of conversing the first sentence of the stimulus into sufficient and necessary condition in order to solve out this type of strengthen question. When I tried to figure this out, I conversed 1st sentence like the following: "Having an efficient, attractive subway system makes good economic sense" ===> " If (we) have an efficient, attractive...., it makes (us) have good economic sense" But, I couldn't find answer with such S&N condition. So, I tried to reverse both conditions as like the following: ====> "(To) make good economic sense, (we must) have an efficient, attractive subway system." Then, I could smoothly fugue this out. In this light, I'm still confused why my first attempt is wrong. Is it possible to make two different S&N condition from the sentence like that?? If it isn't,please explain why which one is right and which one is wrong. Thanks,

Replies
Create a free account to read and take part in forum discussions.

Already have an account? log in

Naz on January 5, 2015

The first way you diagrammed the sentence is correct.

The argument tells us that having an efficient, attractive subway makes, i.e. RESULT IN, good economic sense.

So, we diagram it:

P1: E & A ==> GES
not GES ==> not E or not A

We are also told that the city should always do what result in "GES," i.e. the city should always do whatever sufficient condition that will lead us to "GES"--which we know is "E & A."

C: PNSC

Well, if new subway cars are required in order for the city to have a subway system that is efficient and attractive, then:

(E): E & A ==> NSC
not NSC ==> not E or not A.

So, we know we must have "E & A," and answer choice (E) explains that if we have "E & A," then we must have "NSC," which is our conclusion.

Thus, answer choice (E) is the correct answer because when it is assumed, it allows the conclusion of the argument to be logically drawn.

Hope that was helpful! Please let us know if you have any other questions.

Batman on January 6, 2015

Thanks a lot^^